跳到主要内容

所有问题

1问题
筛选依据
排序依据
标记为
15 投票
1 回答
710 意见

半稳定滤波向量空间,Tannakian范畴。

假设$k$是一个字段(可能是char=0)。设$(V,F)$是一对,其中$V$是有限维$k$向量空间,$F$是$V$的过滤,由有理数索引,满足:$F^i V。。。
马蒂的用户头像
  • 13.1万